How Does the Gompertz Equation Model Tumor Growth?

Click For Summary
The Gompertz equation models tumor growth with the formula dN/dt=-rN*ln(N/K), where N represents tumor cell count and r, K are positive parameters. Critical points and their stability are analyzed through the function f(N)=-rN*ln(N/K), revealing that the slope of N(t) versus t starts positive, decreases to zero at N=K, and then becomes negative. The solution to the Gompertz equation for initial conditions is complex, particularly when maintaining the constant K, leading to confusion in derivations. Asymptotic stability can be assessed without solving the differential equation by examining the sign of f near N=0 and N=K. Understanding the relationship between constants is crucial for accurate modeling of tumor growth dynamics.
Jamin2112
Messages
973
Reaction score
12

Homework Statement



As long as N isn't too small, the growth of cancerous tumors can be modeled by the Gompertz equation,

dN/dt=-rN*ln(N/K),

where N(t) is proportional to the number of cells in the tumor, and r, K > 0 are parameters.

(a) Sketch the graph f(N)=-rn*ln(N/k) verses N, find the critical points, and determine whether each is asymptotically stable or unstable.

(b) For the initial condition N(0)=N0 (N0>0), solve the Gompertz equation for N(t). Does this agree with your results from (a) in the limit as t -->∞?

(c) Sketch N(t) vs. t for initial conditions (i) 0 < N0 < K, (ii) N0=K, and (iii) N0 > K.

Homework Equations





The Attempt at a Solution



(a)

Below is my sketch. I figured, since r,K,N>0 (can't have negative size of tumor), this is just a natural log function upside down and stretched out a little bit.

screen-capture-3.png


I'm not sure how to tell if it's asymptotically stable. I know that on the graph of N(t) versus t, it will start out with a positive slope; decrease in slope until the slope is zero at N(t)=K; then the slope will gradually become more negative.

(b)

dN/dt=-rN*ln(N/K)
==> dN / (N*ln(N/K) = -rdt
==> d/dN (ln(ln(N/K))) = -rdt
==> ln(ln(N/K) = -rt + C
==> ln(N/K) = e-rt+C = ©e-rt (©=ec. I'm going to merge my constant shizzle while I solve)
==>N/K = e©ert
==>N=N(t)= Ke©ert = ®e(-rt) (®=ke©)


... This is where I'm stuck. I've always been told that I can merge constants as I go along; but doing so in this problem screws everything up because I need K to stay with me!
 
Physics news on Phys.org
Check your derivation : ln(ln(N/K) ) doesn't make sense when N <K. It's better to handle the 3 cases separately.
 
Jamin2112 said:
Sketch the graph f(N)=-rn*ln(N/k) that should be f(N)=-rN*ln(N/k) verses N, find the critical points, and determine whether each is asymptotically stable or unstable.


I'm not sure how to tell if it's asymptotically stable. I know that on the graph of N(t) versus t, it will start out with a positive slope; decrease in slope until the slope is zero at N(t)=K; then the slope will gradually become more negative.

(b)

dN/dt=-rN*ln(N/K)
==> dN / (N*ln(N/K) = -rdt
==> d/dN (ln(ln(N/K))) = -rdt
==> ln(ln(N/K) = -rt + C
==> ln(N/K) = e-rt+C = ©e-rt (©=ec. I'm going to merge my constant shizzle while I solve)
==>N/K = e©ert
==>N=N(t)= Ke©ert = ®e(-rt) (®=ke©)


... This is where I'm stuck. I've always been told that I can merge constants as I go along; but doing so in this problem screws everything up because I need K to stay with me!

Your question suggests to me having been copied to here not quite accurately. At any rate a natural reading of the first sentence above seems to be asking for critical points of f. There is one maximum of f but that is an inflection point of N against t. It doesn't come to me to think stability is something you characterise inflection points by.

Confusion between f and N has made your red sentence all wrong but I am sure you will get it right when you realize that.

For the asymptotic stability you do not need to solve the d.e., you just need to look at the sign of f, i.e. of dN/dt, near N=0, and near the top limit N=K (both sides of that one).

For your problem with the arbitrary constant it is a bit fiddly.

You got to ln(N/K) = Ce-rt (I agree with your integration) and you are probably thinking (N/K) = eCe-rt, ah that's N = KeCe-rt my K seems to have been swallowed into a lumped arbitrary constant. But it hasn't - eCe-rt is not eCe-rt but (eC)e-rt, not the same as multiplying K by eC, think about it.
 
Question: A clock's minute hand has length 4 and its hour hand has length 3. What is the distance between the tips at the moment when it is increasing most rapidly?(Putnam Exam Question) Answer: Making assumption that both the hands moves at constant angular velocities, the answer is ## \sqrt{7} .## But don't you think this assumption is somewhat doubtful and wrong?

Similar threads

  • · Replies 1 ·
Replies
1
Views
2K
  • · Replies 13 ·
Replies
13
Views
3K
Replies
2
Views
1K
Replies
5
Views
2K
  • · Replies 10 ·
Replies
10
Views
6K
  • · Replies 5 ·
Replies
5
Views
3K
  • · Replies 5 ·
Replies
5
Views
2K
  • · Replies 24 ·
Replies
24
Views
3K
Replies
1
Views
3K
Replies
1
Views
8K